Difference between revisions of "2006 AMC 12A Problems/Problem 8"

(Solution 2)
(Solution 2)
Line 33: Line 33:
 
<cmath>15 = \frac{n}{2} \cdot (2a + n - 1)</cmath>
 
<cmath>15 = \frac{n}{2} \cdot (2a + n - 1)</cmath>
 
<cmath>2an + n^2 - n = 30</cmath>
 
<cmath>2an + n^2 - n = 30</cmath>
 +
<cmath>n^2 + n(2a - 1) - 30 = 0</cmath>
 +
 +
Now we need to find the number of possible sets of values of a, n which satisfy this equation. Now <math>a</math> cannot be 15 as we need 2 terms. So a can only be less the 15.
 +
 +
Trying all the values of a from 1 to 14 we observe that <math>a = 4</math> is the only option which gives a real solution.
 +
Putting <math>a = 4</math> the equation becomes
 +
 +
<cmath>n^2 + 7n - 30 = 0</cmath>
 +
<cmath>n^2 + 10n - 3n - 30 = 0</cmath>
 +
<cmath>n(n+10)-3(n+10) = 0</cmath>
 +
<cmath>(n-3)(n+10) = 0</cmath>
  
 
== See also ==
 
== See also ==

Revision as of 01:44, 13 January 2024

The following problem is from both the 2006 AMC 12A #8 and 2008 AMC 10A #9, so both problems redirect to this page.

Problem

How many sets of two or more consecutive positive integers have a sum of $15$?

$\textbf{(A) } 1\qquad \textbf{(B) } 2\qquad \textbf{(C) } 3\qquad \textbf{(D) } 4\qquad \textbf{(E) }  5$

Solution 1

Notice that if the consecutive positive integers have a sum of $15$, then their average (which could be a fraction) must be a divisor of $15$. If the number of integers in the list is odd, then the average must be either $1, 3,$ or $5$, and $1$ is clearly not possible. The other two possibilities both work:

  • $1 + 2 + 3 + 4 + 5 = 15$
  • $4 + 5 + 6 = 15$

If the number of integers in the list is even, then the average will have a $\frac{1}{2}$. The only possibility is $\frac{15}{2}$, from which we get:

  • $15 = 7 + 8$

Thus, the correct answer is $\boxed{\textbf{(C) }3}.$


Question: (RealityWrites) Is it possible that the answer is $4$, because $0+1+2+3+4+5$ should technically count, right?

Answer: (IMGROOT2) It isn't possible because the question asks for positive integers, and this means that negative integers or zero aren't allowed.

Note to readers: make sure to always read the problem VERY carefully before attempting; it could mean the difference of making the cutoff.

Solution 2

Any set will form a arithmetic progression with the first term say $a$. Since the numbers are consecutive the common difference $d = 1$.

The sum of the AP has to be 15. So,

\[S_n = \frac{n}{2} \cdot (2a + (n-1)d)\] \[S_n = \frac{n}{2} \cdot (2a + (n-1)1)\] \[15 = \frac{n}{2} \cdot (2a + n - 1)\] \[2an + n^2 - n = 30\] \[n^2 + n(2a - 1) - 30 = 0\]

Now we need to find the number of possible sets of values of a, n which satisfy this equation. Now $a$ cannot be 15 as we need 2 terms. So a can only be less the 15.

Trying all the values of a from 1 to 14 we observe that $a = 4$ is the only option which gives a real solution. Putting $a = 4$ the equation becomes

\[n^2 + 7n - 30 = 0\] \[n^2 + 10n - 3n - 30 = 0\] \[n(n+10)-3(n+10) = 0\] \[(n-3)(n+10) = 0\]

See also

2006 AMC 12A (ProblemsAnswer KeyResources)
Preceded by
Problem 7
Followed by
Problem 9
1 2 3 4 5 6 7 8 9 10 11 12 13 14 15 16 17 18 19 20 21 22 23 24 25
All AMC 12 Problems and Solutions
2006 AMC 10A (ProblemsAnswer KeyResources)
Preceded by
Problem 8
Followed by
Problem 10
1 2 3 4 5 6 7 8 9 10 11 12 13 14 15 16 17 18 19 20 21 22 23 24 25
All AMC 10 Problems and Solutions

The problems on this page are copyrighted by the Mathematical Association of America's American Mathematics Competitions. AMC logo.png